Đến nội dung

Hình ảnh

$f(m+1)\mid (f(n)+1)$

- - - - - 100hamso

  • Please log in to reply
Chủ đề này có 1 trả lời

#1
namcpnh

namcpnh

    Red Devil

  • Hiệp sỹ
  • 1153 Bài viết

Bài toán 44 : Tìm tất cả các hàm $f:\mathbb{N}\rightarrow \mathbb{N}$ thỏa mãn tồn tại $k\in \mathbb{N}$ và 1 số nguyên tố $p$ sao cho $\forall n\geq k$ , $f(n+p)=f(n)$ và nếu $m\mid n$ thì $f(m+1)\mid (f(n)+1)$.

 

Giải :

 

Từ $f(n+p)=f(n)$ bằng qui nạp chứng minh được $f(n)=f(n+mp)$ với $m\in \mathbb{N}$

Vậy nên ta có $f(n_0p)=f(q!)$ với $q,n\in \mathbb{N}$ và $q>n_0p\geq k$

Mà $n_0p-1\mid q!\Rightarrow f(n_0p)\mid f(q!)+1$ với lại $f(n_0p)=f(q!)\Rightarrow f(q!)=1$

Khi $q \rightarrow +\infty$ thì $n_i\mid q!,\forall n_i\in \mathbb{N^*}$

Nên $\prod_{i=1}^{t} f(n_i+1)\mid f(q!)+1 \Rightarrow \prod_{i=1}^{t} f(n_i+1)\mid 2$

Do đó ta có $f(n)=1$ có thể tồn tại $c<k$ mà $f( c )=2$ còn $f(1),f(0)$ có giá trị tự nhiên bất kì  :)

Nếu $k=1$ thì $f(1)=1$, $k=0$ thì $f(n)=1,\forall n\in \mathbb{N}$


Bài viết đã được chỉnh sửa nội dung bởi namcpnh: 08-06-2013 - 12:32

Cùng chung sức làm chuyên đề hay cho diễn đàn tại :

Dãy số-giới hạn, Đa thức , Hình học , Phương trình hàm , PT-HPT-BPT , Số học.

Wolframalpha đây


#2
Idie9xx

Idie9xx

    Sĩ quan

  • Thành viên
  • 319 Bài viết

Bài toán 44 : Tìm tất cả các hàm $f:\mathbb{N}\rightarrow \mathbb{N}$ thỏa mãn tồn tại $k\in \mathbb{N}$ và 1 số nguyên tố $p$ sao cho $\forall n\geq k$ , $f(n+p)=f(n)$ và nếu $m\mid n$ thì $f(m+1)\mid (f(n)+1)$.

Từ $f(n+p)=f(n)$ bằng qui nạp chứng minh được $f(n)=f(n+mp)$ với $m\in \mathbb{N}$

Vậy nên ta có $f(n_0p)=f(q!)$ với $q,n\in \mathbb{N}$ và $q>n_0p\geq k$

Mà $n_0p-1\mid q!\Rightarrow f(n_0p)\mid f(q!)+1$ với lại $f(n_0p)=f(q!)\Rightarrow f(q!)=1$

Khi $q \rightarrow +\infty$ thì $n_i\mid q!,\forall n_i\in \mathbb{N^*}$

Nên $\prod_{i=1}^{t} f(n_i+1)\mid f(q!)+1 \Rightarrow \prod_{i=1}^{t} f(n_i+1)\mid 2$

Do đó ta có $f(n)=1$ có thể tồn tại $c<k$ mà $f( c )=2$ còn $f(1),f(0)$ có giá trị tự nhiên bất kì :)

Nếu $k=1$ thì $f(1)=1$, $k=0$ thì $f(n)=1,\forall n\in \mathbb{N}$ :))


Bài viết đã được chỉnh sửa nội dung bởi Idie9xx: 06-06-2013 - 07:42

$\large \circ \ast R_f\cdot Q_r\cdot 1080\ast \circ$





Được gắn nhãn với một hoặc nhiều trong số những từ khóa sau: 100hamso

0 người đang xem chủ đề

0 thành viên, 0 khách, 0 thành viên ẩn danh